Need help finding roots for a complex number using angles

Click For Summary
The discussion focuses on finding the roots of the complex equation x³ = √(3) - i by converting it to a vector form with a magnitude of 2 and an angle of -30 degrees. The user initially calculates two roots but becomes confused about the existence of multiple angles yielding different complex numbers. It is clarified that while adding multiples of 360 degrees to the angle produces different representations, only three unique roots exist within the interval [0, 360°). The misunderstanding stemmed from not recognizing that these additional angles correspond to the same root in the complex plane. Ultimately, the user gains clarity on the concept of roots in complex numbers.
fishspawned
Messages
66
Reaction score
16
Member warned that the homework template is required
so i am starting with the equation x3 = √(3) - i

first : change to a vector

magnitude = √[ (√(3))2 + 12] = 2
and angle = tan-1( 1/√(3) ) = 30 degrees
(in fourth quadrant)

so i have a vector of 2 ∠ - 30

so i plot the vector on the graph and consider that :

1. the fundamental theorum of algebra tells me i must have three roots.
2. multiplying two complex roots written as vectors will result in the magnitudes multiplied and the angles added together

so to get the root, i simply find the cube root of the magnitude and divide the angle by three

this gets me two initial answers:

3√(2) ∠ - 10
and
3√(2) ∠ 110

the question now comes up with the third root, because I am assuming there should be only one more. But I'm getting MANY more. Since the magnitude remains as 3√(2) then i simply need to find an angle that can be divided into three to get -30 (or 330) degrees.

well : if i take (360 + 330)/3 = 130, it works
if i take (-30 - 360)/3 = -110 , it works.
in fact any addition of 360 works, so I can only guess that these are not really roots. But they do produce entirely different complex numbers - all of them.

I have been assuming there are only three possible roots. Where is my logic going wrong here? Please help. I would greatly appreciate a little enlightenment
 
Physics news on Phys.org
fishspawned said:
so i am starting with the equation x3 = √(3) - i

first : change to a vector

magnitude = √[ (√(3))2 + 12] = 2
and angle = tan-1( 1/√(3) ) = 30 degrees
(in fourth quadrant)

so i have a vector of 2 ∠ - 30

so i plot the vector on the graph and consider that :

1. the fundamental theorum of algebra tells me i must have three roots.
2. multiplying two complex roots written as vectors will result in the magnitudes multiplied and the angles added together

so to get the root, i simply find the cube root of the magnitude and divide the angle by three

this gets me two initial answers:

3√(2) ∠ - 10
and
3√(2) ∠ 110

the question now comes up with the third root, because I am assuming there should be only one more. But I'm getting MANY more. Since the magnitude remains as 3√(2) then i simply need to find an angle that can be divided into three to get -30 (or 330) degrees.

well : if i take (360 + 330)/3 = 130, it works
(360 + 330)/3 = 230. It is the angle of the third root.
fishspawned said:
if i take (-30 - 360)/3 = -110 , it works.
(-30 - 360)/3 = -130, which is equivalent to 230, the previous result.
fishspawned said:
in fact any addition of 360 works, so I can only guess that these are not really roots. But they do produce entirely different complex numbers - all of them.

I have been assuming there are only three possible roots. Where is my logic going wrong here? Please help. I would greatly appreciate a little enlightenment

If you add any times 360° to the angle of the vector, you get the same vector. So you have only three different roots, and usually they are chosen form the interval [0, 360°) .
 
ha! a simple addition mistake threw me completely off. thank you so much. everything makes sense now and i can walk away from this .content
 
Question: A clock's minute hand has length 4 and its hour hand has length 3. What is the distance between the tips at the moment when it is increasing most rapidly?(Putnam Exam Question) Answer: Making assumption that both the hands moves at constant angular velocities, the answer is ## \sqrt{7} .## But don't you think this assumption is somewhat doubtful and wrong?

Similar threads

  • · Replies 22 ·
Replies
22
Views
893
  • · Replies 22 ·
Replies
22
Views
2K
  • · Replies 27 ·
Replies
27
Views
2K
  • · Replies 17 ·
Replies
17
Views
3K
  • · Replies 3 ·
Replies
3
Views
3K
  • · Replies 3 ·
Replies
3
Views
2K
  • · Replies 5 ·
Replies
5
Views
2K
  • · Replies 0 ·
Replies
0
Views
2K
  • · Replies 3 ·
Replies
3
Views
2K
  • · Replies 4 ·
Replies
4
Views
2K